Archer Review 3

Lakukan tugas rumah & ujian kamu dengan baik sekarang menggunakan Quizwiz!

A 60-year old adult walks into the clinic with a forehead laceration. He was going 45 mph on his motorcycle when he skidded on the gravel and fell off his bike. He isn't sure if he hit his head. After checking for significant bleeding and apparent signs of injury, what is the first intervention the nurse should do? A. Place a c-collar on the patient's neck. [58%] B. Take the patient back to a procedure room to stitch the laceration. [4%] C. Palpate the patient's abdomen to check for internal bleeding. [4%] D. Check the patient's pupils. [34%]

Explanation Choice A is correct. After checking for apparent signs of illness, this patient needs c-spine precautions placed immediately. This patient was going faster than 30 mph on a motorcycle. We are not sure if he was wearing a helmet, but with the mechanism of injury, he could have a possible c-spine injury. This is important in preventing any spinal cord damage from occurring. Choice B is incorrect. Even though this intervention needs to be performed at some point, it is not the most important thing to do. A laceration may need pressure to stop the bleeding, but it can wait to be sutured. Choice C is incorrect. This step comes later in the head-to-toe assessment. The emergency physician will utilize an ultrasound machine to assess for any internal bleeding or complications. Choice D is incorrect. This step comes later in the head-to-toe assessment. This will happen after airway/c-spine, breathing, and circulation are assessed. The pupils should be assessed in this situation because the patient isn't sure if he hit his head on the ground. This could result in possible brain injury, including illness. NCSBN Client Need Topic: Physiological Adaptation, Sub-topic: Alterations in Body Systems

Diabetes insipidus is a potential complication of which of the following procedures? A. Surgical removal of the pituitary gland [87%] B. Reduction of thyroid gland mass [9%] C. Hysterectomy [2%] D. Dilation and curettage [2%]

Explanation Choice A is correct. Any surgery that is near the pituitary gland will present with the risk for diabetes insipidus (DI). This is because the posterior pituitary is the gland that regulates antidiuretic hormone (ADH), and a decreased amount of ADH results in DI. Choice B is incorrect. A reduction of mass on the thyroid gland would not result in an increased risk for diabetes insipidus. Choice C is incorrect. A hysterectomy would not result in an increased risk for diabetes insipidus. Choice D is incorrect. A dilation and curettage would not result in an increased risk for diabetes insipidus. NCSBN Client Need: Topic: Physiological Integrity, Subtopic: Reduction of Risk Potential, Endocrine

The nurse is talking about child abuse to a number of parents. The nurse should include which definition of child abuse in the teaching plan: A. Assault is a threat to do bodily harm to another person. [60%] B. Assault is the application of force to another person without lawful justification. [25%] C. It is a legal wrong committed by one person against the property of another. [4%] D. It is a legal wrong committed against the public that is punishable by state law. [11%]

Explanation Choice A is correct. Assault is a threat or an attempt to do violence to another. Battery means touching an individual in an offensive manner or actually injuring another person. Choice B is incorrect. This is the definition of battery. Choice C is incorrect. Assault pertains to damage against a person, not property. Choice D is incorrect. This is a very broad definition. It does not define assault.

A woman in her 30th week of gestation was brought into the emergency department for falling down a flight of stairs. On evaluation, the physician notes a rigid, board-like abdomen. FHR is 167 bpm; with otherwise, stable vital signs. Which obstetric emergency must be anticipated considering a possible abdominal trauma? A. Abruptio placentae [58%] B. Ectopic pregnancy [3%] C. Placenta previa [8%] D. Massive uterine rupture [30%]

Explanation Choice A is correct. External trauma can lead to abruptio placentae, the complete or partial separation of the placenta from the uterine wall. A sign that concealed hemorrhage has occurred is the rapid increase in uterine size along with rigidity. Choices B, C, and D are incorrect. An ectopic pregnancy occurs when the embryo implants outside the uterine cavity. Placenta previa is a condition where the placenta is implanted in the lower segment of the uterus that either entirely or partially covers the cervical os. A massive uterine rupture happens during labor when the uterine contents are extruded through the uterine wall.

Select the theory of stress and coping that is accurately paired with an example of it. A. Response-based theory of stress: Selye's general adaptation syndrome [34%] B. Response-based theory of stress: Lazarus' model of stress and coping [32%] C. Stimulus-based theory of stress: Selye's local adaptation syndrome [17%] D. Transaction-based theory of stress: Holmes and Rahe's model of stress and coping [17%]

Explanation Choice A is correct. In broad terms, there are three categories of models and theories that describe stress and coping. These models include response-based theories of stress, stimulus-based theories of stress, and transaction-based theories of stress. Selye's general adaptation syndrome is an example of a response-based theory of stress and this theory describes the physiological responses to stress. Choice B is incorrect. Lazarus' model of stress and coping is not a response-based theory of stress, but instead, a transaction-based theory of stress. Choice C is incorrect. Selye's local adaptation syndrome is a response-based theory of stress. Choice D is incorrect. Holmes and Rahe's model is an example of a stimulus-based theory of stress.

Which of the following components should the nurse know to include in her handoff at the end of her shift? A. List of p.r.n (as needed) medications administered [53%] B. Normal assessment findings for the shift [10%] C. A complete history of lab results and interventions since admission [19%] D. All scheduled medications the client receives [19%]

Explanation Choice A is correct. Medications administered as needed should be included in the nursing handoff. The nursing handoff should review the client's condition during the past shift accurately, but quickly. It is important to include important information about the client like what has occurred over the previous change and if any "as needed" (p.r.n) medications were administered. Choice B is incorrect. Normal assessment findings for the shift are not a necessary component of the nursing handoff. The nursing handoff should review the client's condition during the past shift accurately, but quickly. Reviewing all normal assessment findings would not only take too long but is not necessary information. Any changes in assessment findings, abnormal findings, and current problems should be included, but normal assessment findings are not required to cover. Choice C is incorrect. A complete history of lab results and interventions since admission is not a necessary component of the nursing handoff. The nursing handoff should review the client's condition during the past shift accurately, but quickly. Going over a complete history of lab results and interventions since admission would not only take too long but would not be pertinent. Choice D is incorrect. All scheduled medications the client receives are not a necessary component of the nursing handoff. The nursing handoff should review the client's condition during the past shift accurately, but quickly. As needed medications, changes in the client's situation, interventions, and the client's response to such interventions are part of the nursing handoff. NCSBN Client Need: Topic: Effective, safe care environment; Subtopic: Coordinated care; Prioritization, delegation, and leadership

An 11-week pregnant client is complaining to the nurse about her hemorrhoids. The nurse understands that hemorrhoids occur because of pressure on the rectal veins from the bulk of the growing fetus. All of the following are measures to alleviate hemorrhoid pain, except: A. Instruct the client to use mineral oil to soften her stools. [46%] B. Rest in a side-lying position daily. [11%] C. Increase the client's fiber and water intake. [8%] D. Apply a cold compress to the area. [35%]

Explanation Choice A is correct. Mineral oil is contraindicated in pregnancy as it decreases nutrient absorption in the mother. Choice B is incorrect. Sleeping in a side-lying position removes the weight of the fetus on the superior and inferior vena cava, promoting venous return and decreasing venous pressure. Choice C is incorrect. Increasing fiber and water intake promote the formation of bulkier stools. Preventing constipation and relieving rectal pain. Choice D is incorrect. Cold compresses relieve pain by vasoconstriction of the hemorrhoids.

The nurse is caring for a patient who had abdominal surgery three days before their assessment. Which sign is a normal part of wound healing and would not need to be reported to the nurse? A. Serous drainage from the site [64%] B. Warm and tender skin [14%] C. Hardened and erythematic skin [19%] D. Foul-smelling drainage from the site [3%]

Explanation Choice A is correct. Moderate amounts of serous drainage from the surgical site is an expected finding after abdominal surgery. Choices B, C, and D are incorrect. Warm, tender, hard, and/or red skin, as well as foul-smelling discharge, indicate a wound infection and should be reported to the primary health care provider. NCSBN client need Topic: Reduction of risk potential, Potential complications from surgical procedures

The nurse is in the screening room of a women's health clinic. The nurse notices a particular woman complaining of back and leg pain, spotting after intercourse with her husband, and vaginal discharge for the past few months. The nurse suspects: A. Cervical cancer [42%] B. Endometrial cancer [19%] C. Ovarian cancer [7%] D. Vaginitis [32%]

Explanation Choice A is correct. Signs and symptoms of cervical cancer include back and leg pain, spotting between menstrual periods and after intercourse, vaginal discharge, and lengthening of a menstrual period. A pap smear is needed to assess cellular changes (i.e. check for cancerous and precancerous conditions). Choice B is incorrect. Endometrial cancer manifests as menorrhagia (excessive menstrual bleeding), low abdominal pain, backache, and constipation due to pressure from an enlarging mass. A biopsy is needed to confirm the diagnosis. Choice C is incorrect. Initial signs and symptoms of ovarian cancer include the following: an increasing abdominal girth due to ovarian enlargement, constipation due to rectal pressure from the enlarging mass, anemia, vomiting, and cachexia. Choice D is incorrect. A bacterial infection causes vaginitis. Signs and symptoms include pruritus, burning urination, dysuria, dyspareunia, and a foul-smelling vaginal discharge.

A 45-year-old man on the neurology floor can understand instructions but is unable to express himself through talking. Which lobe of the brain controls the expression of speech? A. Frontal lobe [51%] B. Parietal lobe [23%] C. Temporal lobe [18%] D. Occipital lobe [8%]

Explanation Choice A is correct. The expression of speech is controlled by Broca's area in the frontal lobe. Broca's area is in the left hemisphere near the motor strip. Choice B is incorrect. The parietal lobe controls the sensory interpretation area for the opposite side of the body. It also controls the ability to interpret spoken words, the ability to read and write, and the ability to see body images. Choice C is incorrect. The temporal lobe controls primary auditory functions. It also controls Wernicke's area, which is the ability to organize language and understand what someone is saying. It also controls the limbic system and the hippocampus, which control emotions and memory, respectively. Choice D is incorrect. The occipital lobe controls visual processing and visual association. NCSBN Client Need Topic: Physiological Adaptation, Sub-topic: Pathophysiology, Nervous System

When the nurse begins to spray a Salem sump tube, she notices dark-brown gastric drainage. What is the first intervention the nurse should take? A. Perform a hemoccult test on the contents [30%] B. Irrigate the tube and check the returns [26%] C. Remove the tube from suction [10%] D. Check the pH of the gastric aspirate [33%]

Explanation Choice A is correct. The nurse should check hemoccult on the dark-brown gastric drainage to look for blood. A Salem sump tube is a double-lumen nasogastric tube used for suction and irrigation of the stomach. The nurse should frequently monitor the color and amount of gastric drainage. The presence of dark brown drainage may indicate bleeding or blood in the gastrointestinal tract. Often, it indicates partially digested blood. Because this may be the case, the first action is to perform a hemoccult test on the specimen to determine the presence of blood. Choices B and C are incorrect. Nasogastric tube irrigation (Choice B) is performed if ordered by the physician. Irrigation is done when NG tube obstruction is suspected. When it is no longer needed and when ordered by the physician, suction can be discontinued (Choice C), and NG tubes can be removed. These interventions (Choices B and C) do not address "dark brown" gastric drainage. Choice D is incorrect. Checking the pH of the gastric contents determines the gastric acidity but does not address the finding of "dark brown" gastric drainage. Determining the pH of the gastric tube aspirate is a method used to confirm the placement of the nasogastric tube within the stomach. Checking the pH can also help monitor for intestinal advancement of the tube. The pH of gastric aspirate is acidic (1 to 5). A pH of 5.5 or less indicates correct placement (unless the client is on pH altering medications). A pH of 6 or more, indicates intestinal aspirate (intestinal placement or migration of the tube). A more alkaline pH of 7 or more indicates that the tube is in the respiratory tract. In case of such incorrect placement, the tube must be removed. Checking the pH for gastric placement is done following initial insertion of the tube, before administration of each bolus of feed/medications, when there are new or unexplained respiratory symptoms/reduction in oxygen saturation, or when there is evidence of tube displacement. NCSBN Client Need Topic: Physiological Integrity; Subtopic: Reduction of Risk Potential

A female client has been hospitalized for several months after abdominal surgery for a ruptured colon. A colostomy was created, the wound was left open, and allowed to heal through granulation. She is on antibiotic therapy, IV gentamicin, to treat the wound infection. The nurse knows that this drug is ototoxic, therefore what should be implemented? A. Instruct the client to report any ringing of the ears, dizziness, or difficulty hearing. [94%] B. Instruct the client to discontinue the drug at once at the first sign of dizziness. [1%] C. Facilitate audiometric testing to determine if hearing loss is caused by an ototoxic drug or other cause. [2%] D. Teach the client to perform the Valsalva maneuver to equalize middle ear pressure and prevent hearing loss. [3%]

Explanation Choice A is correct. The nurse's priority action is to educate the client on drug side effects. Choices B, C, and D are incorrect. The discontinuation of a drug is not an independent nursing intervention and may cause problems in client care. Audiometric testing only confirms hearing loss but does not determine the potential cause. Valsalva's maneuver will not prevent hearing loss.

You have just received a 4-month-old infant to the recovery room after a repair of a cleft lip. In transferring the infant from the stretcher to the bed, what would be the appropriate position for you to place the infant in? A. Supine with the head of the bed elevated 30 degrees [70%] B. Prone with the head turned to the right [22%] C. Trendelenburg [1%] D. Prone with head elevated slightly [6%]

Explanation Choice A is correct. The supine position is recommended in the immediate postoperative period of the cleft lip repair. Such positioning prevents injury to the suture line. Choices B and D are incorrect. Postoperative interventions for cleft lip repair include preventing suture line injury, maintaining an airway free of the accumulation of secretions and monitoring for airway edema/ narrowing. An infant with cleft lip repair should not be placed prone because prone position exposes the surgical site of the lip to suture line injury. This is in contrast to the cleft palate repair where the recommended guideline is to place the client prone or in side-lying position. Choice C is incorrect. The Trendelenburg places the client on a 15- to 30-degree incline, positioning the legs higher than the head. By using gravity to assist in the filling and distension of the upper central veins (venous return to the heart, preload), Trendelenburg increases the cardiac output. Such a position improves the blood pressure in hypotension and hypovolemia. Other indications of Trendelenburg position is to prevent the air embolism while placing and removing central venous catheters. Additionally, a Durant's maneuver ( left lateral decubitus position and Trendelenburg position) is deployed in managing acute venous air embolism. Learning objective: Understand that the post-operative position in the cleft lip repair differs from that of cleft palate. While cleft palate repair clients may be placed prone, those with cleft lip repair should never be placed prone to prevent suture line injury. NCSBN Client Need Topic: Physiological Integrity, Subtopic: Reduction of Risk Potential

The nurse is reviewing the laboratory results of a patient scheduled for surgery. Which of the following should be reported to the primary health care provider (PHCP)? A. Glycosylated hemoglobin (HbA1c) of 7.2% [14%] B. International Normalized Ratio (INR) of 3.5 [65%] C. Hematocrit (Hct) of 42% [3%] D. Blood urea nitrogen (BUN) level of 5 [18%]

Explanation Choice B is correct. An INR of 3.5 seconds is elevated and needs to be reported because the client may bleed. Choice A is incorrect. The HbA1c is elevated but would not impact a client scheduled for surgery. Choice C is incorrect. The hematocrit of 42% is within normal limits. Choice D is incorrect. A BUN level of 5 is decreased but poses no threat to the client.

The nurse is assessing a client with congestive heart failure. Which physical assessment finding should the nurse expect? A. Intermittent claudication [10%] B. S3 gallop [55%] C. Venous stasis ulcers [7%] D. Widened pulse pressure [28%]

Explanation Choice B is correct. An S3 gallop is an expected finding in heart failure. This is often an early manifestation of heart failure; it and this sound are best auscultated at the apex of the heart. Choices A, C, and D are incorrect. Intermittent claudication is a clinical feature of peripheral arterial disease. This causes a client pain as they ambulate and is relieved with rest. The pain may radiate to the ankle or buttock. Venous stasis ulcers are associated with long-term venous insufficiency; ulcer formed due to edema or minor injury to the limb; typically occurs over the malleolus. A widened pulse pressure (difference between the systolic and diastolic) is a feature of increased intracranial pressure, not heart failure. Additional Info A third heart sound, S3 gallop, is an early diastolic filling sound indicating an increase in left ventricular pressure. This sound is often the first sign of HF. This sound is best auscultated over the apex of the heart. If this sound is auscultated during pregnancy, this is a normal finding.

The nurse is caring for a patient with a tracheostomy. Which of the following items is essential to have at the bedside? A. Air humidifier [7%] B. Inner cannula [67%] C. Nasal cannula oxygen [16%] D. Tracheostomy brush [9%]

Explanation Choice B is correct. An inner cannula of the tracheostomy size and one smaller is necessary to keep at the bedside. This is essential in case the inner cannula becomes dislodged. Choices A, C, and D are incorrect. An air humidifier is recommended for a patient with a tracheostomy but not required. This is useful to help prevent the hardening of the secretions since the oropharynx is bypassed. Nasal cannula oxygen is inappropriate to have at the bedside because the oropharynx is bypassed. To deliver oxygen for a patient with a tracheostomy, a trach collar should be used. A tracheostomy brush is useful for tracheostomy care but not necessary. This brush is only used to loosen up hardened secretions. Additional information: When caring for a patient with a tracheostomy, the nurse should have the necessary safety equipment at the bedside, and this includes - Two extra inner cannulas - one fitting the patient and one smaller size. Obturator Oxygen source/tank

The nurse is implementing orders for a client undergoing a barium enema. Aside from the radiology department, which hospital department should be notified of the procedure? A. The cardiac catheterization department. [10%] B. The dietary department. [46%] C. The nuclear medicine department. [26%] D. The hospital laboratory department. [18%]

Explanation Choice B is correct. As part of the preparations for a barium enema, the client needs to be on NPO for 8 - 10 hours. The dietary department needs to be informed about withholding meals within the NPO period. Choice A is incorrect. The procedure is performed in the radiology department, not the cardiac catheterization department. Therefore the cardiac catheterization department does not need to be informed. Choice C is incorrect. The procedure does not contain any nuclear material; therefore, the nuclear medicine department does not need to be informed. Choice D is incorrect. The laboratory should not be informed as it does not involve the laboratory.

The nurse is caring for a client who has the following clinical data as shown in the exhibit provided. Which action should the nurse take based on the 14:00 vital signs? See the exhibit. A. Reassess the client's vital signs (VS) in one-hour. [2%] B. Notify the primary healthcare physician (PHCP). [55%] C. Perform a physical assessment. [9%] D. Stop the infusion of prescribed D5W. [34%]

Explanation Choice B is correct. At 11:00, the client was given intravenous haloperidol, which subsequently increased the client's pulse and temperature. These changes reflect neuroleptic malignant syndrome (NMS). NMS is brought upon by atypical and typical antipsychotics and is manifested by muscle rigidity, tachycardia, and a fever. The nurse needs to report the 14:00 findings to the provider because of hypertension, progressive tachycardia, and fever. Choice A is incorrect. The client is showing signs of NMS. The priority action is to contact the PHCP. Monitoring the client can follow after. Choice C is incorrect. Performing a physical assessment is not necessary at this time as the nurse has sufficient reason to notify the provider. Choice D is incorrect. The priority action is to report to the PHCP. Stopping the D5W infusion is not immediately necessary.

The nurse is caring for a patient receiving buspirone. Which of the following patient statements would indicate a therapeutic response? A. "I am less depressed and able to spend time with my friends." [26%] B. "My anxiety has lessened, and I have started going out more." [50%] C. "I noticed an improvement in my concentration." [11%] D. "I have been able to fall asleep without any problem." [12%]

Explanation Choice B is correct. Buspirone is a medication indicated for individuals with generalized anxiety disorder. The patient reporting decreased anxiety would indicate a therapeutic effect. Choices A, C, and D are incorrect. Buspirone is not indicated for major depressive disorder (MDD). The patient reporting fewer depressive symptoms would not be applicable for buspirone. Improved concentration would be accomplished by medications such as methylphenidate. Buspirone is not used for patients with insomnia. Additional information: Buspirone is a medication indicated for anxiety disorders. This medication modulates serotonin and does not have a sedative effect when compared to benzodiazepines.

Which form of therapy would most likely be used to treat a group of clients affected by phobias? A. Behavioral psychotherapy [18%] B. Cognitive behavioral psychotherapy [55%] C. Psychoanalysis [10%] D. Cognitive psychotherapy [16%]

Explanation Choice B is correct. Cognitive-behavioral therapy (CBT) is a treatment that combines cognitive psychotherapy and behavioral psychotherapy. CBT is recommended as a first-line treatment approach for specific phobias. The behavioral component of CBT involves repeated exposure to the feared situations and thereby promotes fear reduction. For example, a virtual reality exposure strategy using computer technology can be used to simulate real-life situations (e.g. treating fear of flying with repeated exposure in a flight simulator). The cognitive component of CBT facilitates the client to identify the maladaptive thoughts relating to stressors and replace them with realistic thoughts. For example, a client with a specific phobia of elevators may incorrectly believe there is an extremely high chance of getting stuck while in an elevator. This distorted belief can be addressed with cognitive therapy and thereby reduce fear. CBT is also used for generalized anxiety disorder, panic disorder, eating disorders (anorexia nervosa, bulimia), and obsessive-compulsive disorder (OCD). Choice A is incorrect. Behavioral psychotherapy is useful for patients adversely affected by substance-related disorders and other addictive disorders. Some of the techniques used with behavioral therapy include operant conditioning as put forth by Skinner, aversion therapy, desensitization therapy, modeling, and complementary and alternative stress management techniques. Choice C is incorrect. Psychoanalysis deals with the client's subconscious and focuses on the past and current issues. Psychoanalysis and psychodynamic psychotherapy have been used in treating anti-social personality disorders. Choice D is incorrect. Cognitive psychotherapy is primarily used to treat patients with depression, anxiety disorders, or eating disorders. Cognitive therapy is aimed at altering the client's perspective and attitudes relating to stressors. Learning objective: Cognitive-behavioral therapy combines cognitive and behavioral psychotherapy strategies and is the standard first-line treatment for phobias. NCSBN Client Need: Topic: Psychosocial integrity; Sub-Topic: Behavioral interventions

A patient who has recently been brought to the emergency department after experiencing a very traumatic event appears calm and in total control. The nurse assesses this behavior as which of the following defense mechanisms? A. Projection [10%] B. Denial [52%] C. Rationalization [20%] D. Regression [18%]

Explanation Choice B is correct. Denial is a coping mechanism used to protect a patient from a traumatic experience. A patient in denial will behave as though the trauma never occurred. Choice A is incorrect. Projection is a defense mechanism where the patient takes their personal feelings and places them onto someone else, believing the other person is experiencing the undesired feelings. Choice C is incorrect. Rationalization involves working to find a good reason for something negative occurring. Choice D is incorrect. Regression is a coping mechanism where a patient behaves in a manner reminiscent of an earlier, safe time in their life. NCSBN client need Topic: Psychosocial integrity, coping mechanisms

The nurse is caring for a patient who is receiving prescribed doxorubicin. Which of the following findings would indicate the patient is having an adverse effect? A. Urine discoloration [14%] B. Pulmonary congestion [52%] C. Hirsutism [6%] D. Pruritus [28%]

Explanation Choice B is correct. Doxorubicin is an antineoplastic that is indicated for a variety of cancers. Doxorubicin is highly cardiotoxic, which may cause cardiomyopathy. The cardiotoxicity may cause a decrease in left ventricular ejection fraction, therefore, causing pulmonary congestion and, at worse life-threatening pulmonary edema. The patient may present with leg edema, cough, and worsening shortness of breath. Choices A, C, and D are incorrect. An adverse effect is an unexpected pharmacologic effect that occurs even when the medication is administered correctly. On the other hand, a side effect is an unwanted effect that occurs due to drug therapy. Side effects are usually foreseen by the providers, they do not hamper the treatment plan, they are often mild, and self-resolving. On the other hand, adverse effects are serious, life-threatening, and often necessitate treatment change. Reddish discoloration of urine ( Choice A) is an undesired but harmless and expected effect with Doxorubicin. Doxorubicin is red in color and stains the urine, and even the tears. Patients may get alarmed so, the nurse should educate that the reddish urine discoloration is not from blood. Doxorubicin, like most antineoplastics, may cause alopecia, not hirsutism ( Choice C). Finally, pruritus ( Choice D) is a common side effect of doxorubicin. A side effect like pruritis is self-resolving and does not hamper doxorubicin treatment whereas the occurrence of cardiomyopathy most often necessitates changing the treatment plan. Additional information: Doxorubicin may be indicated for a variety of cancers such as breast, ovarian, and bladder cancer. Like many antineoplastics, this medication is a vesicant and should be administered only through a central line to avoid extravasation. This antineoplastic is highly cardiotoxic, and initiating cardiac monitoring during the infusion is strongly recommended.

The nurse is caring for a client who is receiving prescribed enalapril. Which of the following would be essential to teach the client? The client should immediately report A. dry non-productive cough. [31%] B. swelling of the face, lips, and eyes. [54%] C. alterations in taste. [5%] D. the need for follow-up laboratory work. [10%]

Explanation Choice B is correct. Enalapril is an ACE inhibitor used to treat hypertension and congestive heart failure (CHF). The priority teaching would be about the adverse reaction of angioedema (swelling of the face, lips, eyes) because it may lead to respiratory distress. Choices A, C, and D are incorrect. ACE inhibitors may cause a dry, non-productive cough, alterations in taste, and hyperkalemia. However, the essential teaching point is about angioedema's adverse reaction, which could be life-threatening if left untreated. Additional Info Angiotensin-Converting Enzyme Inhibitors (ACE-I) include medications such as (lisinopril, captopril, and enalapril). These medications are prescribed for hypertension, congestive heart failure, and diabetic nephropathy. A dry cough is common with ACE inhibitors. Hyperkalemia may occur with an ACE inhibitor, and potassium supplementation or potassium-sparing diuretics should be used with caution. Like all ACE inhibitors, lisinopril can cause a dry cough, which is annoying and will not harm the client. Lisinopril (and all ACE inhibitors) may be associated with a decrease in renal function. Thus, the creatinine should be monitored during the therapy.

Which percussion sound would indicate further assessment is needed? A. Dull tone over the spleen [22%] B. Hyperressonance over an adult's lung tissue [46%] C. Flat tone over bone [4%] D. Hyperressonance over a child's lung tissue [27%]

Explanation Choice B is correct. Hyperressonance is an abnormal finding over adult lung tissue. It indicates an abnormal increase in the amount of air present, such as with emphysema. Choice A is incorrect. Soft, short, muffled "dull" sounds are normal over dense organs such as the liver and spleen. Choice C is incorrect. Bones produce a "flat" percussion sound in normal healthy adults. Choice D is incorrect. Adult lung tissue should create a "resonant" sound during percussion, but hyperresonance is a normal finding in children's lung tissue. NCSBN Client Need Topic: Pathophysiology, Subtopic: Skills/procedures

The client with an implantable cardioverter defibrillator (ICD) is at the outpatient department. He is concerned about a shock that his friend felt when they were shaking hands. The nurse can discuss which of the following in response to the patient's concern? A. His friend should have an ECG taken to check if his heart rhythm was affected in anyway. [11%] B. He shook hands with his friend at the exact same time the ICD delivered a shock to restore his rhythm, and that he should not worry about anything. [56%] C. The shock was due to the magnetic field the device emits. [21%] D. He should get checked by the physician. [12%]

Explanation Choice B is correct. If the ICD administers shock, others in physical contact with the patient may feel it but will not be harmed. Choice A is incorrect. If the ICD administers shock, others in physical contact with the patient may feel it but will not be harmed. Choice C is incorrect. The ICD does not produce any magnetic field. Choice D is incorrect. The shock felt by the patient's friend is standard and does not warrant any checkup by a physician.

The school nurse is talking to a group of mothers regarding poison prevention and management. Which statement by the mothers indicates a need for further teaching? A. "I need to properly label the containers of poisonous liquids." [3%] B. "I need to make my child vomit in the instance he ingests gasoline." [56%] C. "I can give my child milk or some water to dilute the poison while I rush him to the hospital." [37%] D. "All poisonous materials should be stored away from children." [4%]

Explanation Choice B is correct. Induction of vomiting when a victim has ingested hydrocarbons is contraindicated. Vomiting may lead to inhalation of the poison, worsening the situation. Choice A is incorrect. This is a correct statement. Proper labeling can help prevent accidental ingestion of poisons at home. Choice C is incorrect. This is a correct statement. Diluting the poison can buy some time in getting the child/victim some needed help. Choice D is incorrect. This is a correct statement. Poisonous materials should always be stored away from children and must be locked.

The nurse is taking care of a client with neuropathic pain. He is already on acetaminophen and gabapentin, which provide no relief. He rates his pain at six on a numerical pain rating scale of 0 to 10. According to the WHO's Three-Step Analgesic Ladder, which of the following is indicated next? A. Ibuprofen plus dexamethasone [11%] B. Tramadol plus acetaminophen and gabapentin [50%] C. Hydromorphone plus gabapentin [21%] D. Hydrocodone plus gabapentin [19%]

Explanation Choice B is correct. On a numerical 0-10 pain scale, a rating of 0 represents no pain, 1-3 represents mild pain, a rating of 4-6 represents moderate pain, and a score of 7-10 represents severe pain. A pain rating of #6 is considered reasonable on a numerical pain 0-10 rating scale. This client has already been given a step 1 medication (acetaminophen) along with an adjuvant agent (gabapentin) but still has moderate pain. According to the Three-Step Analgesic Ladder that was developed by the World Health Organization (WHO), a step 2 analgesic medication (tramadol) is indicated when the client is complaining of moderate pain that is not responding to non-opioid analgesics (acetaminophen) plus adjuvant medications (gabapentin). The WHO's three-step ladder is considered the pain treatment standard and provides a guideline for pain treatment. It presents a multifaceted approach to pain treatment in which opioids are considered secondary and not primary. The idea is not to restrict opioids but use them only as necessary. The ladder recommends maximizing non-opioid measures before moving to opioid agents. Non-opioid actions are grouped into two categories: non-opioid analgesics and adjuvant therapies. Adjuvant therapies include both adjuvant non-analgesic medications as well as non-pharmacologic tests. Examples of adjuvant medications in pain management include gabapentin or duloxetine for neuropathic pain; dexamethasone or bisphosphonates for pain from bone metastases. Adjuvant non-pharmacologic therapies include acupuncture, physical measures, and psychotherapy. In the WHO's three-step ladder, opioids are added to the existing regimen of non-opioid analgesics and adjuvants; they are not supposed to replace the current regimen. This strategy maximizes pain control by using the least amount of opioids possible. Choice B is incorrect. A step 1 medication (ibuprofen) is not the category of analgesic to be used when the client is describing the pain as a #6 (moderate). A step 2 medication on the WHO's ladder must be used. Choice C is incorrect. Hydromorphone is a potent opioid (Step 3, analgesic). A step 3 medication is not indicated before trying a step 2 medication for moderate pain. For severe pain (7 to 10 on the numerical scale), step 3 agents can be given. Choice D is incorrect. Hydrocodone is a potent opioid (Step 3, analgesic). A step 3 medication is not indicated before trying a step 2 medication for moderate pain. For severe pain (7 to 10 on a numerical scale), step 3 agents can be given.

The nurse is preparing to teach a client who was recently diagnosed with Meniere's disease. To help the client reduce the incidence of attacks, the nurse should recommend that the client do which of the following? A. Irrigate their ear with sterile water. [7%] B. Reduce dietary sodium intake. [50%] C. Not use earbuds or headphones. [38%] D. Speak with limited inflections. [4%]

Explanation Choice B is correct. Reducing dietary sodium intake is key in the reduction of attacks associated with Meniere's disease. By reducing sodium, the client will reduce endolymphatic fluid, therefore, reducing the incidence of attacks. Choices A, C, and D are incorrect. Irrigating the ear with sterile water is not recommended to prevent attacks. The inner ear is implicated in Meniere's disease, specifically, the accumulation of endolymphatic fluid. Earbuds and headphones should be used for a client with Meniere's as one of the features of this disease is hearing loss. These devices may assist a client with amplifying their hearing. Speaking with limited speech inflections would have no bearing on this disease. Additional information: Meniere's disease is characterized by excessive endolymphatic fluid. This causes three main features of vertigo, tinnitus and sensorineural hearing loss. Nursing care aims to provide education that should focus on diet medication (low salt, limiting caffeine and alcohol) and adherence to pharmacotherapy, including antiemetics, diuretics, antihistamines, and glucocorticoids. NCSBN Client need: Topic: Physiological Adaptation; Subtopic: Illness Management

Relaxin is a hormone that is released throughout a woman's pregnancy to help prepare her uterine ligaments for the growth of her fetus and uterus. A downside to relaxin is that it may: A. Cause high blood pressure in some women [23%] B. Lead to musculoskeletal injury due to loose ligaments [51%] C. Make urinating more difficult than normal [14%] D. Increase bowel motility [12%]

Explanation Choice B is correct. Relaxin can lead to clumsiness because of increased flexibility and ligament relaxation. This clumsiness increases the risk of musculoskeletal injury. Relaxin may also cause round ligament pain, indigestion, and an increase in the frequency of urination. Choice A, C, and D are incorrect. High blood pressure, difficulty urinating, and increased bowel motility are not associated with relaxin. NCSBN client need Topic: Health Promotion and Maintenance, Ante / Intra / Postpartum

A 15-year-old admitted for status asthmaticus has been stabilized. Which activity would be most appropriate for the client? A. Completing a jigsaw puzzle [13%] B. Talking with friends on the phone [56%] C. Watching basketball on television [25%] D. Putting together a necklace [6%]

Explanation Choice B is correct. Teenagers need an opportunity to interact with peers during their times of sickness to have an outlet to express their concerns. Talking to friends over the phone enables the client to achieve this. Choice A is incorrect. Teenagers need an opportunity to interact with peers during their times of sickness to have an outlet to express their concerns. Completing a jigsaw puzzle does not give the teenager an opportunity to achieve this. Choice C is incorrect. Teenagers need an opportunity to interact with peers during their times of sickness to have an outlet to express their concerns. Watching television does not give the teenager an opportunity to achieve this. Choice D is incorrect. Teenagers need an opportunity to interact with peers during their times of sickness to have an outlet to express their concerns. Arts and crafts do not give the teenager an opportunity to achieve this.

An infant is currently stable but has just been diagnosed with cystic fibrosis. Which of the following would be the priority nursing goal for the family? A. Stabilize the child [21%] B. Provide emotional support [46%] C. Arrange for financial assistance [4%] D. Formulate long-term goals [28%]

Explanation Choice B is correct. The family needs emotional support when a chronic condition is newly diagnosed in a family. The parents need to follow up on genetic counseling, treatment options, prognosis, and resources. Choice A is incorrect. The infant has already been stabilized, so there is no longer a need to adjust care for the infant. Choice C is incorrect. This is a long-term goal for the family. Choice D is incorrect. This is a long-term goal for the family.

A nurse has received orders to administer a RhoGAM injection IM to a postpartum patient. Which situation is NOT a contraindication for administration of this injection? A. Administration to a patient who has a history of a systemic allergic reaction to preparations containing human immunoglobulins. [7%] B. Administration of the injection within 72 hours after delivery. [59%] C. Administration to an Rh-positive female patient. [23%] D. Administration to a patient with an elevated temperature. [11%]

Explanation Choice B is correct. The injection should be given within 72 hours after birth. RhoGAM is administered intramuscularly within 72 hours after birth to prevent sensitization to the Rh factor in an Rh-negative woman with an infant who is Rh-positive. This injection will prevent hemolytic disease in subsequent pregnancies. Each vial of RhoGAM is cross-matched to a specific woman. The nurse must do all appropriate checks for patient identification to avoid an error in administration. Choices A, C, and D are incorrect. Any history of a systemic allergic reaction to human immunoglobulins is a contraindication for the RhoGAM injection. RhoGAM is administered to an Rh-negative female. The injection should be withheld in a patient who has an elevated temperature. NCSBN Client Need Topic: Safe and Effective Care Environment, Subtopic: Coordinated Care, Postpartum Procedures

Which of the following nursing diagnoses is the most appropriate for your client who is adversely affected by Addison's disease? A. At risk for fluid overload related to Addison's disease [18%] B. At risk for physical injuries related to Addison's disease [29%] C. At risk for impaired health maintenance related to Addison's disease [36%] D. At risk for muscular rigidity related to Addison's disease [16%]

Explanation Choice B is correct. The nursing diagnosis that is the most appropriate for your client who is adversely affected with Addison's disease is, "at risk for physical injuries related to Addison's disease" because clients with Addison's disease are affected with muscular weakness and fatigue, both of which place the client at risk for injuries like falls, for example. Choice A is incorrect. Clients with Addison's disease are adversely affected by several nursing diagnoses, but they are not affected by the risk of fluid overload. Instead, they are at risk for decreased fluid volumes as a result of Addison's disease. Choice C is incorrect. Clients with Addison's disease are adversely affected by several nursing diagnoses. Still, they are not concerned with a risk for impaired health maintenance because there is currently no evidence of this associated with Addison's disease. Choice D is incorrect. Clients with Addison's disease are adversely affected by several nursing diagnoses, but they are not affected by the risk for muscular rigidity. Instead, they are at risk for muscular weakness as the result of Addison's disease.

The nurse in the emergency department is caring for a patient with chest pain. He is on a cardiac monitor and oxygen at 2 liters/minute. Suddenly, the patient becomes unresponsive, and the nurse sees the rhythm on the monitor. See the exhibit. The correct sequence for the nurse to follow in this scenario is: A. Start CPR, administer epinephrine, continue CPR [18%] B. Start CPR, attach defibrillator, shock, resume CPR, shock, resume CPR, administer epinephrine, continue CPR [55%] C. Attach defibrillator, shock, start CPR, shock, administer epinephrine, continue CPR [18%] D. Monitor the patient until the code team arrives [9%]

Explanation Choice B is correct. The patient is in ventricular tachycardia. The nurse should recognize this rhythm and should immediately start CPR and call for help. When the team arrives with the defibrillator, a team member should attach the pads to the patient and perform immediate defibrillation (shock). Following the collapse, the team should resume CPR for another 2 minutes. After two minutes, evaluate the rhythm if the rhythm is still shockable (ventricular tachycardia or ventricular fibrillation, deliver another shock. Immediately resume CPR while another team member prepares to administer epinephrine. After 2 minutes of CPR, stop CPR for evaluation of the rhythm. Provide another shock if the rhythm is still shockable, then resume CPR and consider the administration of amiodarone. Choices A, C, and D are incorrect. Choice A would be correct for asystole or PEA, but not for pulseless ventricular tachycardia. A cardiac arrest rhythm should always be treated first with high-quality CPR, so choices C and D are not correct. NCSBN Client Need Topic: Management of Care, Sub-Topic: Establishing Priorities; Cardiovascular

A 24-year old woman presents to the emergency department and appears as shown in the exhibit. What type of injury does this assessment finding suggest? See the exhibit. A. CSF leak [1%] B. Basilar skull fracture [59%] C. Brown-sequard syndrome [10%] D. Subarachnoid hemorrhage [29%]

Explanation Choice B is correct. This picture represents a clinical assessment symptom called Raccoon's eyes (retroorbital ecchymosis). Pooling of blood surrounding the eyes is most often associated with fractures of the anterior cranial fossa or basilar skull fracture. This assessment finding may be delayed by 1 to 3 days following the initial injury. If bilateral, this sign is highly suggestive of a basilar skull fracture. Other signs of basilar skull fractures include hemotympanum (pooling of blood behind the tympanic membrane) and Battle sign (retro auricular or mastoid ecchymosis). Choice A is incorrect. Although Cerebrospinal fluid (CSF) leak could be a later complication of this injury, the initial injury that this patient is suffering from is a basilar skull fracture. A CSF leak occurs in about 20% of patients following a basilar skull fracture. When meningeal structures are damaged by the fractured bones, CSF can leak through the subarachnoid space and manifests as "clear fluid" draining from the nostrils (CSF rhinorrhea) or ears (CSF otorrhea). To confirm that the fluid is indeed CSF, the physician may order testing the fluid for beta-transferrin. Choice C is incorrect. The Brown-Sequard syndrome (BSS) is a hemisection of the spinal cord and does not cause Raccoon's eyes. Symptoms of the BSS include weakness and loss of proprioception on one side of the body (ipsilateral side of injury) and loss of temperature sensation on the opposite side. Causes for the BSS include a spinal cord tumor, trauma, ischemia, or infectious or inflammatory diseases (tuberculosis, or multiple sclerosis). Choice D is incorrect. Signs of a subarachnoid hemorrhage (SAH) include severe headache (often stated by the patients as "the worst headache of their life"), photophobia, nausea, vomiting, and vision changes. Causes of SAH include aneurysmal rupture or trauma. NCSBN Client Need Topic: Physiological Integrity, Subtopic: Neurologic

A patient in the prenatal clinic has stated her intention to choose formula feeding for her infant. Identify which action by the nurse is most appropriate in being a patient advocate. A. Remind the patient of why breast feeding is the best method of infant feeding. [17%] B. Request a referral to the lactation consultant. [9%] C. Determine the patient's knowledge base related to infant feeding options. [61%] D. Accept the patient's decision without further discussion [14%]

Explanation Choice C is correct. A central concept of patient advocacy is ensuring that the patient's decisions are based on sufficient information and understanding while supporting the patient's right to exercise autonomy. Choice A is incorrect. This answer does not serve to support the patient's right to autonomy. Choice B is incorrect. A referral to the lactation consultant is not necessarily indicated. Choice D is incorrect. While the nurse should support the patient's choice, it is essential to confirm that the patient's decision-making process is based on adequate information. Bloom's Taxonomy - Analyzing

You are a registered nurse who is performing the role of a case manager in your hospital. You have been asked to present a class to newly employed nurses about your role, your responsibilities, and how they can collaborate with you as the case manager. Which of the following is a primary case management responsibility associated with reimbursement that you should include in this class? A. The case manager's role includes the organization of wide performance improvement activities. [20%] B. The case manager's role includes complete, timely, and accurate documentation. [17%] C. The case manager's role in terms of the clients' being at the appropriate level of care. [46%] D. The case manager's role in terms of contesting denied reimbursements [17%]

Explanation Choice C is correct. A failure to ensure the appropriate level of care jeopardizes reimbursement. For example, care in an acute care facility will not be reimbursed when the client's current needs can be met in a subacute or long-term care setting. RN case managers have a primary case management responsibility associated with reimbursement because they are responsible for ensuring the patient is cared for at the appropriate level, consistent with medical necessity and current patient needs. Choices A, B, and D are incorrect. Nurse case managers do not have organization-wide performance improvement activities, the supervision of complete, timely, and accurate documentation, or challenging denied reimbursements in their role. These roles and responsibilities are typically assumed by quality assurance/performance improvement, supervisory staff, and medical billers, respectively. NCSBN Client Need Topic: Safe and Effective Care Environment, Subtopic: Management of Care

You are the nurse in a pediatrician's office. An 8-year-old boy with a history of asthma is brought to the office with complaints of a drippy nose, congestion, and runny eyes. The NP sees the patient and makes the diagnosis of allergic rhinitis. The NP prescribes an intranasal corticosteroid and an intranasal antihistamine. The outcome from allergic rhinitis that would put this child at highest risk is: A. Impaired sleep [12%] B. Decreased school performance [7%] C. An asthma attack [65%] D. Irritability [15%]

Explanation Choice C is correct. An asthma attack would put this child at the highest risk. Studies show that uncontrolled allergic rhinitis can make asthma much more challenging to manage. Since this child has a history of asthma, he should be treated aggressively to manage allergic rhinitis. Choices A, B, and D are incorrect. Although impaired sleep, decreased school performance, and irritability are all risks associated with allergic rhinitis, asthma is a life-threatening illness that must be treated quickly. Anything that interferes with that treatment (such as allergic rhinitis) should be avoided. NCSBN Client Need Topic: Health Promotion and Maintenance, Sub-Topic: Health Promotion/Disease Prevention, Respiratory

The nurse receives laboratory results for several clients under her care. Which client result would the nurse report to the physician immediately? A. An elevated amylase result on a client diagnosed with acute pancreatitis. [8%] B. An elevated WBC count on a client with a septic leg wound. [11%] C. A urinalysis showing many bacteria for a client receiving chemotherapy. [57%] D. A serum glucose of 235 mg/dl in a client with type 1 diabetes mellitus [24%]

Explanation Choice C is correct. Chemotherapy places the client at an increased risk for infection by disrupting mucosal barriers and by suppressing the immune system. A urinalysis result of many bacteria would indicate to the nurse that there is an ongoing urinary tract infection in the client, and this would warrant her to notify the physician immediately to start antibiotic therapy. Choice A is incorrect. An elevated amylase result in a client diagnosed with acute pancreatitis is an expected finding, and this should not warrant immediate referral to the physician. Choice B is incorrect. An elevated WBC count in a client diagnosed with a septic leg wound is an expected finding. WBCs usually increase in response to inflammation, infection, trauma, or stress. This should not warrant an immediate referral to the physician. Choice D is incorrect. An above-normal blood glucose level is a common finding in a client with type I diabetes mellitus; this does not necessitate immediate referral to a health care provider. Some poorly controlled type I diabetes mellitus patients may present with ketoacidosis. Choice D did not mention any acidosis. When answering prioritization questions, do not assume a threatening finding to be present when it's not stated in the question's vignette. Although the blood glucose needs to be better controlled, above-average serum glucose of 235mg/dl without any evidence of ketoacidosis is not an immediate threat to the client.

A patient presents with a round, non-tender nodule on the left wrist that is more pronounced upon flexion. The nurse would recognize this as which of the following conditions? A. Olecranon bursitis [12%] B. Bouchard node [33%] C. Ganglion cyst [47%] D. Pilar cyst [8%]

Explanation Choice C is correct. Ganglion cysts are common, benign tumors over a tendon sheath or joint capsule. They are typically non-tender unless the tumor puts pressure on a nerve. When on the wrist, they become more noticeable with flexion. A ganglion cyst generally resolves on its own and does not require treatment, but can be drained/removed if causing discomfort. Choice A is incorrect. Olecranon bursitis is a common form of bursitis that occurs at the tip of the elbow. It typically presents as a large, soft, red, painful nodule due to inflammation of the bursa. Choice B is incorrect. A Bouchard node refers to a hard, non-tender bony overgrowth on the proximal interphalangeal joint, commonly seen in osteoarthritis. Choice D is incorrect. A pilar cyst is a fluid-filled cyst that originates in a hair follicle. Pilar cysts are commonly found on the scalp. NCSBN Client Need Topic: Adult health - Musculoskeletal, Subtopic: Alterations in body systems, pathophysiology

The emergency department is packed with victims from a capsized ship. Almost all of the victims are suffering from hypothermia. The staff is busy treating their hypothermia with active warming methods. All of the following are active external rewarming methods, except: A. Immersing the client in a 40°C bath. [41%] B. Placing the client on a warming blanket. [2%] C. Infusing warmed IV fluids. [45%] D. Placing radiant lamps over the client. [12%]

Explanation Choice C is correct. Infusing warmed IV fluids is an example of active core warming, not an external rewarming. Therefore, this is the correct answer to the question. Choices A, B, and D are incorrect. Immersing the client in a 40°C bath, covering the client with a warming blanket, and placing lamps over the client are all methods of active warming.

Which part of the laryngeal cartilage is a full circular ring and is the narrowest part of the airway in young children? A. Hyoid [26%] B. Arytenoid [8%] C. Cricoid [56%] D. Thyroid [10%]

Explanation Choice C is correct. The cricoid appears as a full circular ring and is the most narrow part of the airway. While intubating, it can be useful to place pressure on the cricoid to make the airway more comfortable to access. Choice A is incorrect. The hyoid is a semi-circle ring, not a circular ring. It helps support the tongue. Choice B is incorrect. The arytenoid muscle is at the back of the larynx and allows the vocal cords to work correctly. Choice D is incorrect. The thyroid is an organ that sits below the "Adam's apple" and is not a part of the airway. NCSBN Client Need Topic: Physiological Adaptation, Sub-topic: Pathophysiology, Respiratory System

A registered nurse is giving staff assignments for the day. Which client is the most appropriate assignment for a nursing assistant? A. A client requiring a colostomy change [10%] B. A client receiving continuous total parenteral nutrition [1%] C. A client who needs assistance with urine specimen collections [74%] D. A client with impaired swallowing but is allowed to take food by mouth with the aid of a thickener [15%]

Explanation Choice C is correct. The nurse should be able to match the skills of the staff member to the needs of the clients. In this case, the nursing assistant is best assigned to the client needing assistance with urine specimen collection. Choices A, B, and D are incorrect. These clients are not appropriate assignments for a nursing assistant. Colostomy changes and TPN must not be assigned to unlicensed personnel. The client with impaired swallowing is at risk for aspiration and will, therefore, need a lice

The nurse is taking care of a client receiving a D5LR intravenous infusion. Suddenly, the client complains of chest pain and difficulty breathing. On exam, there is cyanosis and tachycardia. The nurse also notices an empty IV bottle. What is the initial intervention of the nurse? A. Replace the empty IV bottle with a new one. [4%] B. Replace the IV line and attach a new IV bottle. [1%] C. Stop the IV infusion and turn the client on his left side with the head of the bed lowered. [50%] D. Stop the IV infusion and notify the physician. [44%]

Explanation Choice C is correct. The nurse should suspect an air embolism because the patient is presenting with characteristic symptoms in a setting where the fluid infusion is complete and the IV drip set is still open. An empty IV "bottle" offers this clue. Manifestations of an air embolism include tachycardia, hypotension, chest pain, difficulty breathing, and cyanosis. Air embolism may cause blockage of small pulmonary vessels compromising the gas exchange, obstruction of ventricular pumping, and arrhythmias. In practice, the replacement of IV infusion bottles with collapsible airbags has largely minimized the risk of air embolism during IV infusions. The nurse's initial action would be to turn off the infusion system, place the client on his left side with the head lowered (left Trendelenburg position), and then notify the physician. The left-sided Trendelenburg position will help the air bubble float in the right ventricle/right atrium and prevents it from causing right ventricular outlet obstruction. Choice A is incorrect. Replacing the IV bottle is not an appropriate intervention in this situation since the client is already presenting with signs and symptoms of air embolism. To prevent air embolism, the nurse should have replaced the IV bottle before it is empty. At this time, the nurse should turn off the infusion system and place the client in the left-Trendelenburg position. Choice B is incorrect. Replacing the IV line and hooking up a new IV bottle does not address the air embolism. Choice D is incorrect. A nurse should never delay a life-saving intervention that is within the scope of his/her practice. After stopping the infusion, the next immediate action for the nurse would be to place the client on his left side with the head lowered to trap the air in the right atrium. After this, the nurse must notify the physician.

The senior nurse is evaluating a newly registered nurse who is assisting a 40-year-old status-post left knee replacement surgery to ambulate using a cane. Which action by the new nurse would warrant additional instructions from the senior nurse? A. The nurse stands at the client's left side during ambulation. [8%] B. The nurse instructs the client to move the cane at the same time with the left leg. [25%] C. The nurse instructs the client to hold the cane with his left hand. [53%] D. The nurse instructs the client to hold the cane 4 -6 inches from his foot. [14%]

Explanation Choice C is correct. This is an inaccurate action of the nurse (therefore, the correct answer to the question) and requires further instruction from the senior nurse. The client should be instructed to hold the cane with his right hand (the hand on the unaffected side) when moving. Choices A, B, and D are incorrect. The new nurse does not need further instruction for these appropriate actions. The nurse should stand by the affected side of the client during ambulation in case the client loses his balance or strength (Choice A). The client should move the cane together with his affected leg to provide support and balance (Choice B). The cane should be positioned 4-6 inches from his foot (Choice D).

The nurse observes a newly hired nurse caring for a client. It would require immediate intervention if the nurse observes the newly hired nurse A. checks the physician's order for a blood transfusion and ensures that it is complete. [1%] B. verifies the client's name, blood compatibility, and expiration date with another nurse. [4%] C. spikes lactated ringers to infuse with the blood product. [19%] D. instructs the unlicensed assistive personnel (UAP) to obtain baseline vital signs. [75%]

Explanation Choice C is correct. This is an incorrect action by the nurse, therefore, the correct answer to the question. Only standard saline solution is used in flushing blood products as other IV fluids cause hemolysis. Choice A is incorrect. This is a correct action by the nurse. The nurse should ensure that the physician's order is complete before starting the transfusion. Choice B is incorrect. This is a correct action by the nurse. The nurse should double-check the order, client number, and identification, blood compatibility, and expiration date with another nurse as the most common cause of ABO incompatibility reactions is human error. Choice D is incorrect. This is a correct action by the nurse. The nurse instructing the UAP to obtain baseline vital signs is appropriate to delegate.

The nurse is caring for a patient with a breast tumor. The patient reports some trouble breathing upon investigation. The nurse sees that the patient has a puffy face/neck, nasal congestion, and a raspy voice. The nurse would most likely consider which of the following disorders that could be occurring in this patient? A. Spinal cord compression [6%] B. Non-Hodgkin's Lymphoma (NHL) [34%] C. Superior vena cava syndrome [52%] D. Shock [8%]

Explanation Choice C is correct. This patient's tumor is found in the chest and thus may obstruct blood flow to and from the superior vena cava. Frequent clinical presentations of superior vena cava syndrome include blurred vision, hoarse voice, stridor, dyspnea, and nasal congestion. Choices A, B, and D are incorrect. These do not explain the patient's presentation. NCSBN client need Topic: Physiologic Adaptation, Medical Emergencies

The nurse is caring for a patient about to undergo adrenalectomy to treat pheochromocytoma. Which of the following assessments is a priority in the pre-operative period? A. Intake and Output (I&O) [8%] B. Blood urea nitrogen level (BUN) [14%] C. Vital signs [53%] D. Level of consciousness (LOC) [25%]

Explanation Choice C is correct. Vital signs are the most critical priority in the pre-operative period. Vital signs provide a baseline for the health care professionals as well as help monitor for hypertension, a complication of pheochromocytoma. Choice A is incorrect. Monitoring intake and output are essential but more important in the post-operative period of adrenalectomy. Choice B is incorrect. Measuring the patient's BUN level is not a priority in the pre-operative period. Choice D is incorrect. Evaluating the patient's level of consciousness is essential, but not as crucial as getting baseline vitals before performing adrenalectomy. NCSBN client need Topic: Physiological Integrity, Reduction of Risk Potential

What is the correct documentation of the patient's peripheral pulse when the finding is that the posterior tibial pulse is weak and thready? A. Grade C posterior tibial pulse [9%] B. Posterior tibial pulse is Grade B [13%] C. The client's posterior tibial is 2 [19%] D. Posterior tibial pulse is 1 [59%]

Explanation Choice D is correct. When assessing pulses, the strength, volume, and fullness of the peripheral pulses are categorized and documented as follows: 0: Absent pulses 1: Weak pulse 2: Normal pulse 3: Increased volume 4: Abounding pulse Choices A and B are incorrect. Grades and grading are not used to document pulses. Choice C is incorrect. The pulse is weak and thready, not regular. NCSBN Client Need Topic: Physiological Integrity, Subtopic: Vital Signs

The nurse is caring for a group of clients. It is a priority to follow up on which client situation? A. A client admitted with an asthma exacerbation that is wheezing while receiving albuterol via nebulizer. [37%] B. A client admitted with pulmonary emphysema who puts on their nasal cannula oxygen before eating. [7%] C. A client with pneumonia is ambulating around the nursing unit while wearing a surgical mask. [7%] D. A client receiving oxygen via nonrebreather and has an oxygen saturation of 92%. [49%]

Explanation Choice D is correct. A client receiving oxygen via non-rebreather is receiving approximately 80%-95% Fio2. If the best oxygen saturation is 92%, this is concerning and may warrant more aggressive measures to improve oxygen saturation. Choices A, B, and C are incorrect. A client admitted with an asthma exacerbation would have manifestations such as tachypnea and wheezing. This is not a priority because the prescribed treatment is being administered. A concern would be if the wheezing does not improve or if the sudden cessation of wheezing should occur. A client with pulmonary emphysema should eat while using a nasal cannula as this increases the oxygen demand. A client with pneumonia should be encouraged to ambulate around the nursing unit. The appropriate PPE is applied to this client, which is a surgical mask. Additional Info Nonrebreather is an oxygen delivery device that may deliver the highest oxygen level in the low-flow systems. This device features a reservoir bag that allows the client to draw the needed oxygen. Additionally, flaps prevent room air from entering the exhalation ports. If a client is not responding favorably to this therapy, the primary healthcare provider (PHCP) may consider intubation and mechanical ventilation. When nonrebreather therapy is initiated, the oxygen delivery device should be set at 10 to 15 L/min to ensure that the bag is kept inflated.

The nurse is discussing cardiac dysrhythmias with a student. Which of the following statements, if made by the student, would indicate effective understanding? A. "Atrial fibrillation may cause venous thromboembolism." [17%] B. "Immediate defibrillation is needed for asystole." [16%] C. "Cardioversion may be used for ventricular fibrillation." [14%] D. "Adenosine is used to treat supraventricular tachycardia." [53%]

Explanation Choice D is correct. Adenosine is the initial medication utilized to manage supraventricular tachycardia (SVT). This medication slows the conduction time through the AV node and restores normal sinus rhythm. Choices A, B, and C are incorrect. Atrial fibrillation does lead to a potential thrombus that may evolve into an embolism; however, it does not cause venous thromboembolism because the blood pooling in the left atrial appendage gets ejected into the arterial circulation, not venous. Thus, the most likely complication is an embolic stroke. Asystole is treated with high-quality CPR and IV epinephrine. Defibrillation is not indicated for asystole. Ventricular fibrillation responds to defibrillation, not cardioversion. Defibrillation delivers a higher amplitude of joules and does not need to be synchronized. Cardioversion can only be done if the client has a pulse. Additional Info Supraventricular tachycardia (SVT) is a tachycardia that is caused by an impulse recirculating throughout the atrial pathway. Adenosine is used to treat SVT; before giving this medication, the nurse must have the client connected to a cardiac monitor and have a defibrillator as well as a crash/code cart nearby. The client should have the pads affixed to them just in case the medication causes a prolonged period of asystole. The medication must be pushed by rapid IV push along with an immediate flush of 0.9% saline.

Which of the following is least likely to influence the potential for a client to comply with lithium therapy after discharge? A. The impact of lithium on the client's energy level and lifestyle. [10%] B. The need for consistent blood level monitoring. [17%] C. The potential side effects of lithium. [10%] D. The client's friend's opinion on his need to take medication. [63%]

Explanation Choice D is correct. Although a patient's social network may influence terms of compliance, this influence is typically secondary when compared to the other factors listed. Lithium is believed to alter the activity of neurons containing dopamine, norepinephrine, and serotonin by influencing their release, synthesis, and reuptake. Therapeutic actions include stabilization of mood during periods of depression. It is neither antimanic nor antidepressant in individuals without bipolar disorder. Choice A is incorrect. Clients who are on lithium therapy are usually greatly affected by lithium, which can be a significant determinant of compliance. Choice B is incorrect. Many patients become non-compliant, especially after an extended period, due to the frequency of required physician and lab visits. Choice C is incorrect. Side effects of lithium are another deterrent to compliance. Potential side effects include fine tremors, drowsiness, diarrhea, and weight gain. NCSBN Client Need Topic: Physiological Integrity, Subtopic: Pharmacological Therapies

The nurse is caring for a client at the first prenatal visit. The primary healthcare provider (PHCP) has prescribed testing for syphilis. The nurse anticipates which laboratory testing? A. Brain Natriuretic Peptide (BNP) [4%] B. Comprehensive Metabolic Panel (CMP) [11%] C. Complete Blood Count (CBC) [32%] D. Rapid Plasma Reagin (RPR) [53%]

Explanation Choice D is correct. An RPR is a common screening test for syphilis infections. This test is often confirmed with a fluorescent treponemal antibody absorption (FTA-ABS) test. Choices A, B, and C are incorrect. A BNP test is utilized to assist in the diagnosis of heart failure. A CMP is testing that may detect problems with the liver or any other electrolyte abnormalities. A CBC is testing that may reveal disorders associated with blood dyscrasias. NCLEX Category: Reduction of Risk Potential Activity Statement: Diagnostic Tests Question type: Knowledge/comprehension

The nursing student is explaining the cause of Cushing's disease. Which of the following statements indicate a correct understanding of this illness? A. Cushing's disease occurs when insulin is overproduced. [5%] B. Cushing's disease is the result of the underproduction of aldosterone. [17%] C. Cushing's disease occurs when androgen hormones are underproduced. [5%] D. Cushing's disease is the result of the increased production of pituitary hormones. [73%]

Explanation Choice D is correct. Cushing's disease occurs when adrenocorticotropic hormones are over secreted by the pituitary gland, increasing cortisol. Choice A is incorrect. The overproduction of insulin does not characterize Cushing's disease. Choice B is incorrect. The underproduction of corticosteroid hormones is Addison's disease. Choice C is incorrect. Cushing's disease is unrelated to the overproduction of androgen. NCSBN client need Topic: Physiological Integrity, Physiologic Adaptation

Which nursing diagnosis would be the highest priority for a patient with a medical diagnosis of Legionnaire's disease? A. Disturbed body image [7%] B. Impaired skin integrity [13%] C. Risk for infection [20%] D. Ineffective airway clearance [60%]

Explanation Choice D is correct. Legionnaire's disease refers to a type of pneumonia caused by the Legionella bacteria, typically found in water or soil. The most appropriate and highest priority nursing diagnosis would be ineffective airway clearance, as this disease impairs the airway and lung function. Choice A is incorrect. Disturbed body image would not be expected to be a priority for this patient in terms of Legionnaire's disease diagnosis, as it affects the airway and lungs. Choice B is incorrect. Impaired skin integrity would not be expected to be a priority for this patient in terms of Legionnaire's disease diagnosis, as it affects the airway and lungs. Choice C is incorrect. Risk for infection would not be an appropriate nursing diagnosis for this patient, as the infection is already present.

The charge nurse is observing a newly hired nurse care for a patient who sustained a closed head injury, is receiving mechanical ventilation, and is at risk for developing increased intracranial pressure (ICP). Which of the following actions, if performed by the newly hired nurse, would require intervention by the charge nurse? A. Suctioning the patient when the high-pressure alarm sounds. [31%] B. Hyperventilating with 100% FiO2 prior to suctioning. [5%] C. Performs oral care with a chlorhexidine solution. [9%] D. Maintaining the head of the patient's bed more than 90 degrees. [55%]

Explanation Choice D is correct. Maintaining a patient's head of the bed more than 90 degrees is detrimental for a patient with a traumatic brain injury ( TBI). The patient should avoid hip and neck flexion as this raises intracranial pressure ( ICP). While elevating the head end of the bed beyond 30 degrees may drop the ICP further, it can also cause an unwanted drop in the mean arterial pressure ( MAP). A decrease in MAP reduces cerebral perfusion pressure ( CPP). A fall in CPP is detrimental to the patient with a TBI. Therefore, the head of the bed recommendation for a patient with a risk for increased ICP is 30 to 45 degrees. Such an angle decreases the ICP while maintaining adequate CPP around 70 to 80 mm Hg. Such head of the bed elevation is also necessary to prevent the patient from developing ventilator-acquired pneumonia. Choices A, B, and C are incorrect. Suctioning is a necessary procedure for patients with artificial airways. Suctioning the patient when the high-pressure alarm sounds is an appropriate intervention. A high-pressure alarm is triggered when an obstruction is evident in the tubing. Not intervening immediately may cause airway compromise and put the client at risk of death. While suctioning may cause an increase in the ICP, one should use the ABC prioritization method and address the airway first. There is no absolute contraindication to suctioning when clinical indicators indicate the need for it. When clinically warranted, a patient should be hyperventilated with 100% oxygen before suctioning. Oral care with chlorhexidine or hydrogen peroxide is recommended to prevent ventilator-acquired pneumonia. Learning Objective When considering head elevation in patients with increased ICP, one should also ensure adequate CPP is maintained. A 30-45 degree elevation decreases ICP while also maintaining the CPP. Elevating the head of the bed beyond 45 degrees may drop the CPP. Additional Info When caring for a patient with a TBI, the nurse should maintain a low stimulating environment. The patient should be positioned with the neck midline with their body. The earliest indicator of a patient having increased ICP is alterations in level of consciousness.

Which of the following images shows the most appropriate position for an infant who has just had a cleft palate surgery?

Explanation Choice D is correct. Most appropriate position for child status-post cleft palate surgery, is prone. Due to the unique anatomy in cleft-palate, these children are at an increased likelihood of their tongue falling back into their airway, causing obstruction and respiratory distress. The prone position prevents the tongue from falling backward. A prone position is also recommended to facilitate the drainage of excessive secretions post-operatively. Choice A is incorrect. This is Trendelenburg's position. It is not recommended in children with a cleft palate due to the risk for airway obstruction caused by the tongue falling back into the airway. Choice B is incorrect. This is a right lateral recumbent position. Lateral positioning can prevent the tongue from obstructing the airway and it can be used in infants with cleft palate during feeding and sleep. However, this is not the best position post-surgically. Post-operatively, the goal is to facilitate the drainage of excessive secretions. The prone position is most helpful to serve that purpose. Supine and side-lying positions are preferred post-operative positions in cleft lip; not the cleft palate. Choice C is incorrect. This is a supine position. It is not recommended in children with a cleft palate due to their risk for airway obstruction caused by their tongue falling back into the airway. Supine and side-lying positions are preferred post-operative positions in cleft lip; not the cleft palate.

The nurse is caring for a client in premature labor receiving terbutaline infusion. All of the following manifestations would alert the nurse to stop the infusion, except: A. Blood pressure of 91/58 mmHg [14%] B. Heart rate of 132 beats/minute [14%] C. Serum potassium level of 3.3 mEq/L [17%] D. Blood glucose level of 130 mg/dL [55%]

Explanation Choice D is correct. Terbutaline may increase the blood glucose level. The nurse should monitor the client's blood sugar levels while on this drug. However, this client's blood sugar level is at 130 mg/dL, an acceptable value to continue Terbutaline infusion. Choice A is incorrect. Terbutaline is a beta-agonist. By acting on beta-2 receptors in vascular smooth muscle, it causes vasodilation. Hypotension is a common side effect of Terbutaline. The nurse should stop the infusion when the blood pressure drops below 90/60 mmHg. Choice B is incorrect. By acting on beta-1 receptors in the heart, Terbutaline increases the heart rate (tachycardia). Tachycardia is also a common side effect of Terbutaline. The nurse should stop the infusion when the heart rate is more than 120 bpm. Choice C is incorrect. Hypokalemia is another common side effect of Terbutaline. The client's potassium is only 3.3 mEq/L. A serum potassium level less than 3.5 mEq/L is considered hypokalemia. Hypokalemia should alert the nurse to stop the terbutaline infusion.

Which of the following questions is least useful in the assessment of a client who is requesting an AIDS test? A. Are you a drug user? [8%] B. Do you have many sex partners? [6%] C. What is your method of birth control? [35%] D. How old were you when you became sexually active? [51%]

Explanation Choice D is correct. The age at which sexual activity began is not relevant as it does not usually provide information that identifies the presence of risk factors for AIDS. When interviewing clients who are seeking/receiving care, the nurse needs to be nonjudgmental. Also, many patients are reluctant to disclose illicit drug use unless a safe environment has been established. Use words that describe behaviors like "drinking" and "using cocaine" instead of words that describe people such as junkie, crack head, addict, alcoholic, and other pejorative terms. Start with less threatening questions. Examples include: What over-the-counter and prescription medications are you taking? Do you inject any of those? How often do you use alcohol? Tobacco? When was the last time you used a drug from a non-medical source? Do not assume anything. Drug use occurs in all socioeconomic strata. If a patient says they use or have used drugs, ask about specific medications (e.g. marijuana, heroin, methamphetamine). Don't forget that people also inject insulin, steroids, and hormones. Sharing injection equipment with these medications can also create the risk of HIV transmission. Choice A is incorrect. Drug use is a risk factor for AIDS. Choice B is incorrect. Multiple sex partners are a risk factor for AIDS. Choice C is incorrect. Although birth control methods may not prevent the spread of AIDS, using effective birth control is essential to help prevent pregnancy and the risk of having a baby born HIV positive. Additionally, patients should be educated that using a condom is the best way to prevent AIDS/HIV transmission through sex. NCSBN Client Need Topic: Safe and Effective Care Environment, Subtopic: Basic Care and Comfort, Therapeutic Communication

The nurse is in charge of a male client scheduled for a liver biopsy at 8 AM. In preparing this client for the procedure, the nurse should do which of the following? A. Inform the client that he will be kept NPO for 24 hours before the biopsy. [15%] B. Let the client practice holding his breath for 1 minute. [14%] C. Inform the client that he will be receiving a laxative to prevent bowel distention as this can apply pressure on the liver. [23%] D. Inform him that his vital signs will be monitored closely after the procedure. [47%]

Explanation Choice D is correct. The client will be monitored closely for bleeding and shock after the procedure. It is appropriate to monitor vital signs. Choices A, B, and C are incorrect. The preparation for a liver biopsy does not include placing the client on NPO, nor administration of a laxative. The client will be asked to hold his breath but only for 5-10 seconds.

An infant is returned to the unit after a surgical repair of a cleft lip on the left side. Which of the following positions should the nursing staff place this child in? A. On their stomach [5%] B. Prone [16%] C. On the left side [13%] D. On the right side [66%]

Explanation Choice D is correct. The infant should be placed on the right side. Post-surgical complications include vomiting and aspiration. Lateral positioning prevents aspiration. This infant should be placed on the right side so as not to disturb the surgical site on the left side. A cleft lip is a congenital malformation in which soft tissue or bone fails to fully connect while the infant develops in utero. Cleft-lip may be associated with cleft palate but can occur as an isolated abnormality. Choice A is incorrect. Infants should not rest or sleep on their abdomen due to the risk of sudden infant death syndrome ( SIDS). The "Back to Sleep" campaign emphasizes this to raise awareness that infants should be put to sleep on their back, not on the abdomen. The only exception to this guideline is in the clients who have undergone cleft palate repair. In case of "cleft palate" repair; a prone position or lying on the abdomen is acceptable to facilitate drainage of secretions and prevent airway obstruction. Choice B is incorrect. Prone is not an appropriate position for an infant who has just had "cleft lip" surgery. If placed prone, the suture line over the lip may be rubbed against the surface in post-cleft lip cases. However, this differs from the cleft palate position. Laying prone after "cleft palate" surgery is appropriate and preferred because it helps drain secretions reduces the risk of airway obstruction and prevents the tongue from falling backward. Choice C is incorrect. While lateral positioning is appropriate for an infant who has just had cleft lip surgery, this infant had surgery on their left side. Laying on the side of the surgery is not appropriate for patient safety. NCSBN client need Topic: Physiological Integrity, Reduction of Risk Potential

A client is currently experiencing bradycardia, low blood pressure, and dizziness. Which of the following does the nurse expect to be ordered? A. Defibrillation [6%] B. Digoxin [23%] C. Monitor the client closely [22%] D. Prepare patient for transcutaneous pacing [49%]

Explanation Choice D is correct. The normal heart rate in an average adult is between 60 to 100 beats per minute. A heart rate less than 60 beats per minute is referred to as bradycardia. Bradycardia can be symptomatic or asymptomatic. Some healthy adults and athletes may have a heart rate between 40 and 60 beats per minute and do not experience any symptoms. When symptomatic, bradycardia can lead to shortness of breath, dizziness, and low blood pressure (hypotension, shock). A patient experiencing symptomatic bradycardia will likely need transcutaneous pacing. In addition, an EKG must be performed to confirm the rhythm. The etiology of bradycardia may vary and include reversible (medications) and irreversible causes (heart blocks). Therefore, one should explore causes, but the priority intervention in a patient experiencing symptoms from bradycardia is to restore the heart rate quickly with transcutaneous pacing and maintain circulation. Choice A is incorrect. Defibrillation is recommended when the patient is experiencing pulseless ventricular tachycardia or ventricular fibrillation. Choice B is incorrect. Digoxin is a cardiac glycoside that has negative chronotropic action on the sinus node. Therefore, digoxin decreases the heart rate and would be dangerous in this patient with symptomatic bradycardia. Choice C is incorrect. While this patient should be monitored closely, priority action (transcutaneous pacing) should quickly restore the heart rate. NCSBN client need | Topic: Physiological Integrity, Reduction of Risk Potential

The tips on a walker: A. Must be replaced every other day to prevent wear and tear that places the client at risk for injury. [7%] B. Must be inspected by the engineering or biomedical engineering department of the facility. [17%] C. Have been proven to be highly dangerous in recent years because they slide on the floor. [4%] D. Wear away at varying rates according to the surface being walked on and the frequency of its use. [71%]

Explanation Choice D is correct. The tips on a walker wear out at varying rates according to the surface being walked on and the frequency of its use. For this reason, the tips should be inspected before each application or perhaps every day to ensure that these tips show no signs of wear and tear that could place the client at risk for injury. Choice A is incorrect. The tips on a walker do not have to be replaced every other day to prevent wear and tear that places the client at risk for injury, but the tips should be inspected before each use or perhaps every day to ensure that these tips show no signs of wear and tear that could place the client at risk for injury. Choice B is incorrect. The tips on a walker do not have to be inspected by the engineering or biomedical engineering department of the facility; they can be checked by the nursing staff. Choice C is incorrect. The tips on a walker have not been proven to be highly dangerous in recent years because they slide on the floor; the tips on a walker protects the clients' safety.

You are administering IV magnesium to a patient with a magnesium level of 1.5 mEq/L. You check on them halfway through the infusion, and they report that their face feels flushed. What is the priority nursing intervention? A. Slow down the infusion rate. [35%] B. Notify the healthcare provider. [2%] C. Reassess the patient when the infusion finishes. [4%] D. Stop the infusion. [58%]

Explanation Choice D is correct. This is the priority nursing action and should be completed first. The infusion should immediately be turned off to minimize any further reaction. As soon as the injection is stopped, the provider can be notified. In priority nursing questions, the priority will always be whatever action you can take that immediately helps the patient. In this question, turning off the infusion will help stop/prevent further reactions to the magnesium infusion, so it is the priority. Choice A is incorrect. It is inappropriate to slow down the infusion rate simply. The patient feeling flushed and warm indicates a high magnesium level and possibly a reaction to the infusion. Their magnesium level could have increased dramatically in response to the injection leading to an increased risk for seizures and arrhythmias. If the reaction is mild, the provider may recommend slowing down the infusion, but this is not the appropriate nursing intervention. Choice B is incorrect. While it is appropriate to notify the healthcare provider of this change, there is another nursing action listed that takes priority and should be carried out first. Choice C is incorrect. It is inappropriate to continue the infusion and reassess the patient after the injection. They are reacting and immediate action is needed. NCSBN Client Need: Topic: Physiological Integrity, Subtopic: Pharmacological therapies, Electrolytes

An 82-year-old man presents to the emergency department after a ground-level fall. The paramedics tell you that the left pupil was fixed and dilated. Upon arrival, the patient's elbows, wrists, and fingers are flexed, and legs extended and rotated inward. What is the most important intervention for this patient? A. Obtain IV access immediately [30%] B. Turn patient on his side [10%] C. Obtain accurate history from the family [2%] D. Take him straight to the CT scan [57%]

Explanation Choice D is correct. This patient's left pupil is fixed and dilated, which means it is not reactive to light and stays the same size. When this happens, it can be clinically inferred that there is a lesion or hemorrhage on the opposite (contralateral) side of the brain. The patient also exhibits decorticate (flexor) posturing, with elbows, wrists, and fingers flexed, while the legs are extended and rotated inward. Often, such abnormal posturing indicates severe brain damage. The patient sustained a fall and these symptoms likely represent raised intracranial pressure due to intracranial hemorrhage. This patient needs to be taken straight to the CT department to obtain a CT scan of the brain. This will allow the physician to diagnose the patient and initiate early treatment. Choice A is incorrect. Even though obtaining IV access is an important intervention, it is not the priority at this time. A non-contrast CT scan is usually the first intervention to detect a hemorrhage. Intravenous contrast is not necessary. The nurse can obtain IV access after the urgent CT scan is performed. Early diagnosis and appropriate treatment are critical in these settings. Choice B is incorrect. If the patient started having a seizure, then he would need to be turned onto his side. However, he is posturing, which is not a seizure. There are two different types of posturing; decorticate and decerebrate. Decorticate looks as if the patient is turning his or her arms into the core of the body. Decerebrate looks like the patient's arms are facing outwards, away from the body. Choice C is incorrect. This intervention is important, especially to understand any events before arriving at the hospital, medications taken, and recent procedures completed. NCSBN Client Need Topic: Physiological Adaptation; Sub-topic: Alterations in Body Systems

The nurse is conducting client and family education about dietary considerations related to Parkinson's disease. One priority consideration that the nurse should highlight in teaching is to address the risk of: A. Too much fluid and drooling [9%] B. Loss of appetite and aspiration [32%] C. Loose stools and choking [5%] D. Difficulty swallowing and constipation [55%]

Explanation Choice D is correct. With Parkinson's disease, eating problems include dysphagia, aspiration, constipation, and the risk of choking. Choices A, B, and C are incorrect. Drooling is a symptom of Parkinson's disease; however, it does not take priority over aspiration and dysphagia. Fluid overload, diarrhea, and loss of appetite (anorexia) are problems not directly related to Parkinson's disease.

Which drug is considered the antidote for methamphetamine? A. Naloxone [29%] B. Acetylcysteine [7%] C. Atropine [6%] D. Flumazenil [16%] E. None [43%]

Explanation Choice E is correct. Unfortunately, there is no known antidote for methamphetamine. Treatment is supportive only. Choice A is incorrect. Naloxone is the antidote for opioid overdose. Choice B is incorrect. Acetylcysteine is the antidote for acetaminophen overdose. Choice C is incorrect. Atropine is the antidote for organophosphate overdose or poisoning. Choice D is incorrect. Flumazenil is the antidote for benzodiazepine overdose. NCSBN Client Need: Topic: Physiological Integrity, Subtopic: Physiological Adaptation

You are teaching a group of new graduate nurses about the long term effects of congestive heart failure. You know that they understand your teaching when they state the following expected findings. Select all that apply. A. Polycythemia [21%] B. Clubbing [39%] C. Pulsus alternans [27%] D. Macewens sign [12%]

Explanation Choices A and B are correct. A is correct. Polycythemia is defined as an abnormally increased concentration of hemoglobin in the blood; it is a serious long term effect of congestive heart failure. It is due to the effects of chronic hypoxia on the body. The body senses the decrease in oxygen and increases its production of red blood cells in order to carry more oxygen to the body. The problem is that there is no more oxygen available, so the body continues to be hypoxic, and continues to produce red blood cells in an attempt to correct this. After a while of overproducing red blood cells, the blood becomes very thick. B is correct. Clubbing is defined as a bulbous enlargement of the ends of the fingers or toes. It is a sign of chronic hypoxia. Patients who experience congestive heart failure over long periods of time often experience clubbing in their fingertips due to the lack of oxygen reaching their distal extremities over time. Choice C is incorrect. Pulsus alternans is a physical finding with an arterial pulse waveform showing alternating strong and weak beats. It is suggestive of left ventricular systolic impairment. It is not a finding specific to chronic periods of hypoxia, rather it represents weak heart function. Choice D is incorrect. McEwan's sign is not a sign of chronic hypoxia, rather it is a sign used to detect hydrocephalus. The examiner percusses on the skull near the junction of the frontal, temporal, and parietal bones and can auscultate a "cracked pot", or hyper-resonant sound if hydrocephalus is present. McEwan's sign is not related to congestive heart failure or chronic hypoxia. NCSBN Client Need: Topic: Psychosocial Integrity; Subtopic: Pediatrics - Cardiac

Which of the following statements regarding growth in the infant are true? Select all that apply. A. Infants should triple their birth weight by 12 months of age. [46%] B. Infants should double their birth weight by 6 months of age. [47%] C. Infants should triple their birth weight by 6 months of age. [2%] D. Infants should double their birth weight by 12 months of age. [4%]

Explanation Choices A and B are correct. Infants should triple their birth weight by 12 months of age (Choice A). Infants should double their birth weight by six months of age (Choice B). Choice C is incorrect. Infants should triple their birth weight by 12 months of age, not six months of age. If an infant has already tripled their birth weight at six months of age, this represents too much weight gain and could pose a potential problem. A nutritionist and special infant care team should be consulted. Choice D is incorrect. Infants should double their birth weight by six months of age, not 12 months of age. If it takes an infant 12 months to increase their birth weight, this indicates that they are not growing fast enough, and there may be issues related to their growth and development. A nutritionist and special infant care team should be consulted. NCSBN Client Need: Topic: Effective, safe care environment, Subtopic: Coordinated care; Pediatric Growth and Development

Which of the following types of grief are not considered normal and require some interventions from the members of the healthcare team? Select all that apply. A. Complicated grief [39%] B. Anticipatory grief [8%] C. Unresolved grief [45%] D. Grief as the result of a perceived loss [9%]

Explanation Choices A and C are correct. Complicated grief and unresolved grief are not considered normal. Both types, therefore, require some interventions from the healthcare team members. Complicated grief is defined as pathological grief; it is characterized by maladaptive coping methods with the loss and the loss of normal functioning six months after the event. Unresolved grief is characterized by an exaggerated and prolonged period of mourning. Choices B and D are incorrect. Anticipatory grief (Choice B) is considered normal; this type of grief occurs when the person reacts to a loss that is anticipated in the future. For example, a woman may have anticipatory grief before the actual loss of the breast with a mastectomy. Grief as the result of a perceived loss (Choice D) is normal. Injuries that occur as the result of both actual and perceived failures must be resolved.

While working in a long-term care facility, you are assigned to a client diagnosed with dementia who is disoriented and combative. The provider has ordered soft wrist restraints for this client. Throughout your shift, you document the proper use of this safety device. This documentation should include which of the following? Select all that apply. A. Reason for use of restraints [31%] B. Date and time order for restraints is received [28%] C. Patient's response to restraints [27%] D. Release from restraints for private bathroom breaks [13%]

Explanation Choices A and C are correct. The reason the restraints are needed must always be documented thoroughly. If the nurse feels that the documented reason is inaccurate or inadequate, she should consult a health care provider to see if other measures or safety devices are more appropriate for the patient. Restraints are always a last resort. Evaluating the patient's response to the restraints is key to the documentation requirements. This helps both the health care providers and nursing teams determine the best method of keeping the patient safe. Choice B is incorrect. The date and time that the order for restraints was received is not relevant documentation. However, the date and time that the restraints were applied to the patient would be considered relevant documentation. Choice D is incorrect. It is required that the restraints are periodically released for exercise and assessments of the skin, circulatory status, and neurovascular status, but it is inappropriate to offer the patient private bathroom breaks while the restraints are released. This puts the patients at risk for harm to themselves or others and should not be allowed. When restraints are no longer indicated, it can be possible to offer the patient more private breaks. NCSBN Client Need: Topic: Safety and Infection Control Subtopic: Use of Restraints/Safety Devices

Which of the following conditions are considered risk factors for postpartum hemorrhage? Select all that apply. A. Microcephaly [7%] B. Dystocia [43%] C. Placenta previa [45%] D. Singleton pregnancy [5%]

Explanation Choices B and C are correct. Dystocia, which is prolonged and painful labor, is a risk factor for postpartum hemorrhage. Prolonged labor, specifically, can dramatically increase the risk of postpartum hemorrhage (Choice B). Placenta previa is a risk factor for postpartum hemorrhage. In placenta previa, the placenta is covering the cervix of the mother rather than sitting in the fundus of the uterus as it should be. This puts the mother at risk for postpartum hemorrhage (Choice C). Choice A is incorrect. Microcephaly is a newborn complication where the newborn is born with a head smaller than average. This is not a risk factor for a woman to experience postpartum hemorrhage. If you selected this answer, you might have gotten it confused with macrosomia, a condition where the infant is larger than average, specifically higher than 4,000 g. This is a risk factor for postpartum hemorrhage. Choice D is incorrect. A singleton pregnancy or a pregnancy with only one fetus does not pose a risk for postpartum hemorrhage. The risk factor for postpartum hemorrhage occurs with multiples, such as twins or triplets. NCSBN Client Need: Topic: Physiological Integrity, Subtopic: Physiological adaptation; Labor and Delivery

While working in the surgery department, a nurse witnesses the patient signing the consent form for his gastric bypass surgery. The surgeon asks the patient if they have any further questions, and the patient says no. After the surgeon leaves, the patient begins asking the nurse questions about the surgery and the possible complications. He states, "I don't know if I should do this. What do you think?". What are the appropriate responses from the nurse? Select all that apply. A. "That is up to you, what do you want to do?" [4%] B. "It sounds like you still have some concerns about the operation, is that correct?" [48%] C. "Let me call the surgeon and have him come back in. He can go over these concerns with you so that you understand everything clearly." [48%] D. "I wouldn't do it, this surgery sounds risky!" [0%]

Explanation Choices B and C are correct. This is a therapeutic response to the patient's concerns as it validates how they are feeling and makes sure the nurse understands what they are saying. If the patient confirms that they have concerns about the operation, the surgeon should be called back to speak with them again. It is the nurse's responsibility to ensure that if the client does not fully understand everything after they have signed the informed consent, the surgeon comes back to clarify. It is the surgeon's responsibility to explain everything regarding the surgery, but it is the nurse's responsibility to verify that they understand. Choice A is incorrect. While it is appropriate to encourage the patient to make their own choices, this is not a therapeutic response. Furthermore, it is the nurse's responsibility to ensure that if the client does not fully understand everything after they have signed the informed consent, the surgeon comes back to clarify. It is the surgeon's responsibility to explain everything regarding the surgery, but it is the nurse's responsibility to verify that they understand. Choice D is incorrect. This is not therapeutic and it is inappropriate for the nurse to offer personal opinions to the patient. NCSBN Client Need: Topic: Management of CareSubtopic: Informed Consent

You are taking care of a 12-year-old boy in the PICU admitted after a suicide attempt. He ingested multiple pills from his mother's medicine cabinet, some of which were oxycodone. He was found down and successfully resuscitated after 10 minutes of CPR. When doing your admission assessment, you notice several scars as well as open cuts on his wrists. What are the appropriate ways to respond to these findings? Select all that apply. A. Talk with his parents and inform them this is not acceptable. [2%] B. Ask him directly how he got these cuts. [44%] C. Tell him that he should never harm himself again. [6%] D. Speak with the patient in a nonjudgmental manner to understand what he feels has happened. [48%]

Explanation Choices B and D are correct. It is essential to address the issue directly. If the nurse does not bring up the problem, the patient will likely try to avoid it (Choice B). It is essential to speak to this patient in a non-judgmental way. This allows the patient to open up and discuss their feelings. They may tell you why they have participated in self-harm activities, allowing you to begin to help them (Choice D). Choice A is incorrect. Telling his parents is not the appropriate nursing action. This does not deal with the issue directly and may cause the patient not to trust the nurse with their true feelings. Choice C is incorrect. Telling the patient not to harm himself again is not therapeutic communication. This negates the patient's feelings and does not help him in any way. NCSBN Client Need: Topic: Psychosocial adaptation Subtopic: Mental health concepts

What are the expected fundal assessment findings for a woman who delivered a set of twins one hour ago via Cesarean section? A. The fundus is hard, midline, and 1-2 fingerbreadths above the umbilicus. [37%] B. The fundus need not be assessed because of the C-section. [12%] C. The fundus is to the right of the umbilicus and soft. [7%] D. The fundus is hard, midline, and at the level of the umbilicus. [44%]

Explanation Choice D is correct. Regardless of the mode of delivery, this is the normal postpartum fundus at one to two hours. Immediately after delivery and expulsion of the placenta, the uterus is about the size of a grapefruit and is located midline in the abdomen, halfway between the umbilicus and the symphysis pubis. Over the next several hours, the fundus will rise on the midline of the stomach to the level of or slightly above the umbilicus. Subsequently, the height of the fundus decreases by at least 1 cm or one fingerbreadth daily as the uterus goes through the process of involution. By the 10th day, the fundus is usually not palpable. Uterine "involution" refers to the return of the uterus to its pre-pregnancy size and condition. Involution begins soon after the expulsion of the placenta and occurs due to the contraction of the uterine smooth muscle. Assessing the fundus (top of the uterus) is a crucial component of post-delivery assessment—a lack of proper uterine involution results in complications such as postpartum hemorrhage. While assessing the fundus following delivery, you must evaluate for: Orientation: This represents the orientation of the fundus in relation to the umbilicus (referred by terms "to the left of the umbilicus", "midline", or "to the right of umbilicus"). Please note "midline" does not refer to the height of the fundus; instead, it refers to a side-to-side orientation in the midline (normal orientation). The fundus should immediately return to midline after delivery. If it deviates from the midline, it indicates a "distended bladder." Height: Assessing the height of the fundus is crucial to determine fundal descent because this is how the uterus returns to its original position in the true pelvis. Height is measured in fingerbreadths or centimeters (cm) in relation to the umbilicus (1 fingerbreadth = 1 to 1.5 centimeters). Please note immediately after delivery, the fundus is "below" the umbilicus- at midway between the symphysis pubis and umbilicus. The fundus then rises to the level of the umbilicus at about 1-2 hours post-delivery. At 12 hours, the fundus rises to 1 fingerbreadth above the umbilicus. Subsequently, it starts descending. In the next few days, the fundus "descends" at a rate of 1 to 2 cm every 24 hours. Around the 6th day, the fundus is located halfway between the symphysis pubis and umbilicus. A week after birth, the uterus returns to the true pelvis. At 10 to 14 days, the fundus should not be palpable. By six weeks, the uterus returns to pre-pregnancy weight. Choice A is incorrect. The fundus is located at the level of or slightly above the umbilicus at 1 to 2 hours after delivery. At 12 hours, it is about one fingerbreadth above the umbilicus. Note that the fundus rises within a few hours after birth before it begins its descent. Choice B is incorrect. The fundus must be assessed, regardless of the mode of delivery. Choice C is incorrect. The fundus should not be to the right of the midline or soft within a few hours after delivery. It should be in the midline. Right or leftward deviation indicates bladder distension. NCSBN Client Need Topic: Physiological Integrity; Subtopic: Reduction of Risk Potential


Set pelajaran terkait

Hip, Pelvis, Thigh and Knee Conditions

View Set

PrepU Chapter 32: Skin Integrity and wound Care

View Set

Cell Bio Chapter 11: Membrane Structure

View Set

Nucleotides the building blocks of DNA

View Set

Advanced Data Structures and Algorithms: Quiz 1

View Set

Anthem - Foundation/Basics Assessment 2022

View Set